Finding the particular solution to an ODE with set boundary conditions.

Click For Summary
The discussion revolves around solving a particular ordinary differential equation (ODE) with given boundary conditions. The initial confusion stemmed from the incorrect auxiliary equation, which was clarified to yield a homogeneous solution of the form x_h = C_1 cos(2t) + C_2 sin(2t). The user successfully differentiated the general solution and applied the boundary conditions to derive constants C_1 and C_2. There was a concern about maintaining real coefficients when substituting complex exponentials, but it was noted that constants can remain complex or real. The thread emphasizes the importance of correctly applying boundary conditions to find the particular solution to the ODE.
Jack_O
Messages
64
Reaction score
0

Homework Statement



80234296.jpg


Homework Equations



N/A

The Attempt at a Solution



5184b1eb.jpg


The problem and attempt are as above, I'm not sure where to go from here though. I'm not sure what to do with the boundary condition of dx/dt=-2 and t=0.
Any help appreciated.
 
Physics news on Phys.org
The answer to your auxiliary equation is wrong, m=+/- 2i not +/- 2 so the homogeneous solution is of the form x_{h}=C_{1}cos(2t)+C_{2}Sin(2t)
 
Thanks for pointing that out, I've had another go and realized i needed to differentiate the general solution and then sub in the other boundaries to get the other simultaneous equation. My completed solution looks like this:

742403f1.jpg


I have also done part c), got the answer x=(i/sqrt3)[-e^(2it)+e^(-2it)]
 
make your solution without complex coefficients so y_{h}=C_{1}sin2t+C_{2}cos2t

then work out the result C_{1}sin0+C_{2}cos0=1
C_{2}=1

y_{h}'=2C_{1}cos2t-2C_{2}sin2t

2C_{1}cos0-2C_{2}0=-2

-2C_{1}-2=-2

C_{1}=0...
 
If i use Euler's formula to sub out e^(2it) for cos(2t)+i*sin(2t) won't i still be left with complex coefficients? Their are complex numbers in the (1+i) term. I don't see how i can cancel out all the complex numbers.
 
Question: A clock's minute hand has length 4 and its hour hand has length 3. What is the distance between the tips at the moment when it is increasing most rapidly?(Putnam Exam Question) Answer: Making assumption that both the hands moves at constant angular velocities, the answer is ## \sqrt{7} .## But don't you think this assumption is somewhat doubtful and wrong?

Similar threads

Replies
3
Views
2K
  • · Replies 1 ·
Replies
1
Views
1K
  • · Replies 8 ·
Replies
8
Views
2K
Replies
6
Views
2K
  • · Replies 1 ·
Replies
1
Views
2K
  • · Replies 1 ·
Replies
1
Views
2K
  • · Replies 8 ·
Replies
8
Views
3K
  • · Replies 4 ·
Replies
4
Views
2K
  • · Replies 4 ·
Replies
4
Views
2K
Replies
0
Views
1K